0 Daumen
4,4k Aufrufe

wäre es möglich, dass ihr mir bei dieser Aufgabe mit der Lösung helfen? ich habe die schön gelöst, aber ich bin mir mit meinen Ergebnissen unsicher. Mit dem Lösungsweg wäre es ja super!

Der Tangenshysperbolicns ist definiert als

$$ \tanh ( x ) : = \frac { \sinh ( x ) } { \cosh ( x ) } $$

Zeige, dass die Ableitung der Umkehrfunktion Artanh : (-1, 1) → ℝ und dass diese durch

$$ \operatorname { Artanh } ^ { \prime } ( x ) = \frac { 1 } { 1 - x ^ { 2 } } \forall x \in ( - 1,1 ) $$

gegeben ist.


IMG_7083.jpg 

Avatar von

2 Antworten

+2 Daumen

ich weiß nicht, ob das so gemeint ist und ob es einfachere geht.

$$\sinh(x)=\frac{1}{2}\cdot (e^x-e^{-x})\\\cosh(x)=\frac{1}{2}\cdot (e^x+e^{-x})\\\tanh(x)=\frac{ (e^x-e^{-x})}{ (e^x+e^{-x})}=y\\z=e^x\\y=\frac{z-z^{-1}}{z+z^{-1}}\\y\cdot ( z+z^{-1})=z-z^{-1}\\yz+yz^{-1}=z-z^{-1}\\-z+yz=-z^{-1}-yz^{-1}\\-z\cdot (1-y)=-z^{-1}\cdot (1+y)\\\frac{-z}{-z^{-1}}=\frac{1+y}{1-y}\\z^2=\frac{1+y}{1-y}\\(e^x)^2=\frac{1+y}{1-y}\\e^{2x}=\frac{1+y}{1-y}\\2x=\ln(\frac{1+y}{1-y})\\x=\frac{1}{2}\cdot \frac{1+y}{1-y}\\\text{x und y vertauschen, damit es dann auch die Umkehrfunktion ist}\\Artanh(x)=\frac{1}{2}\cdot \ln\left(\frac{1+x}{1-x}\right)$$

Jetzt die Ableitung mittels Kettenregel, Quotientenregel und \(\frac{d}{dx}(\ln(x))=\frac{1}{x}\)

Nebenrechnung:

$$\frac{d}{dx}\left(\frac{1+x}{1-y}\right)=\frac{(1-x)+(1+x)}{(1-x)^2}=\frac{2}{(1-x)^2}$$

$$ Artanh'(x)=\frac{1}{\cancel{2}}\cdot \frac{1}{\frac{1+x}{1-x}}\cdot \frac{\cancel{2}}{(1-x)^2}\\=\frac{1-x}{1+x}\cdot \frac{1}{(1-x)^2}=\frac{1}{(1+x)\cdot (1-x)}=\frac{1}{1-x+x-x^2}=\boldsymbol{\underline{\underline{\frac{1}{1-x^2}}}}$$

Hier der Graph.

~plot~ 1/(1-x^2) ~plot~

Da sieht man, dass die Funktion auf dem Intervall I = (-1;1) stetig und auch differenzierbar. Wie man das rechnerisch zeigt, weiß ich nicht ganz.

Ich hoffe, das hilft weiter.

Avatar von 5,4 k
+1 Daumen

berechne erstmal die Ableitung des tanh ( die braucht man später):

cosh(x)*tanh(x)=sinh(x) |  ableiten

sinh(x)*tanh(x)+cosh(x)tanh(x)'=cosh(x)| :cosh(x)

tanh(x)^2+tanh(x)'=1

tanh(x)'=1-tanh(x)^2

Verwende nun die Umkehrregel:

arctanh'(x)=1/tanh'(arctanh(x))=1/(1-tanh(arctanh(x))^2)=1/(1-x^2)

Avatar von 37 k

Besser vielleicht artanh.

Ein anderes Problem?

Stell deine Frage

Willkommen bei der Mathelounge! Stell deine Frage einfach und kostenlos

x
Made by a lovely community